Determine all of the open sets in given product topology

Click For Summary
The discussion focuses on determining the open sets in the subspace topology of Z and the product topology of Z × Y. The correct subspace topology for Z is identified as having the basis {∅, {2}, {2, 3}}. The product topology is generated by the Cartesian product of the bases from Z and Y, leading to a basis with four sets. The initial attempt incorrectly included elements not present in Z, but the revised basis correctly lists the open sets as unions of subsets of the new basis. The final conclusion affirms the understanding of the product topology and its open sets.
sa1988
Messages
221
Reaction score
23

Homework Statement



##X = \{1,2,3\}## , ##\sigma = \big\{\emptyset , \{1,2\}, \{1,2,3\} \big\}##, topology ##\{X, \sigma\}##
##Y = \{4,5\}## , ##\tau = \big\{\emptyset , \{4\}, \{4,5\} \big\}##, topology ##\{Y, \tau\}##
##Z = \{2,3\} \subset X##

Find all the open sets in the subspace topology on ##Z## and determine all the open sets in the product topology on ##Z \times Y##

Homework Equations

The Attempt at a Solution


[/B]
As with previous threads, I'm hoping I'm on the right track. Just looking to check my answers.

Subspace topology on ##Z##, ##\sigma_z = \big\{\emptyset , \{2\}, \{2, 3\} \big\}##

Product topology is generated by basis ##\{ U \times V : U \in \sigma_z , V \in \tau \}##

Thus the product topology on ##Z \times Y## should be created from basis:

##\beta = \big\{\emptyset, \{(2,4)\}, \{(2,4),(2,5)\}, \{(2,4),(2,5),(3,4),(3,5)\}, \{(4,2)\}, \{(4,2),(4,3)\}, \{(4,2),(5,2)\}, \{(4,2),(4,3),(5,2),(5,3)\}\big\} ##

Thus all the open sets in the product topology on ##Z \times Y## are found in the union of all sets of basis elements in ##\beta## , plus the empty set:

## \Big\{\big( \cup U_\lambda \big)## ## \forall## ## U_\lambda \subset \beta\Big\} \cup \{\emptyset\}##

Is this all okay? Thanks.

(Many topology threads incoming - this subject is definitely my kryptonite)
 
Physics news on Phys.org
The subspace topology of ##Z## has a basis with two non-empty sets, and the topology of ##Y## has a basis with two non-empty sets. So the product basis must have ##2\times 2=4## sets. The above list for ##\beta## has seven elements, so cannot be correct. Notice also that the first component of an element of the product set comes from ##Z##, which does not contain ##4##, yet the above list includes open sets containing elements whose first component is ##4##, so they cannot be from ##Z\times Y##.

To get the basis of the product topology, just write out the four elements of the Cartesian product of the two bases.
 
  • Like
Likes sa1988
andrewkirk said:
The subspace topology of ##Z## has a basis with two non-empty sets, and the topology of ##Y## has a basis with two non-empty sets. So the product basis must have ##2\times 2=4## sets. The above list for ##\beta## has seven elements, so cannot be correct. Notice also that the first component of an element of the product set comes from ##Z##, which does not contain ##4##, yet the above list includes open sets containing elements whose first component is ##4##, so they cannot be from ##Z\times Y##.

To get the basis of the product topology, just write out the four elements of the Cartesian product of the two bases.

Ah ok, I think I see where I went wrong. For reasons unknown to me, I took the products ##Z \times Y## and ##Y \times Z##

Second attempt, I'd go for:

##\beta = \big\{ \{(2,4)\}, \{(2,4),(2,5)\}, \{(2,4), (3,4)\}, \{(2,4),(2,5),(3,4),(3,5)\}\big\}##

And then the open sets in that product topology are all possible unions of all possible subsets of ##\beta##, along with the empty set.
 
Yes, that looks right!
 
  • Like
Likes sa1988
Question: A clock's minute hand has length 4 and its hour hand has length 3. What is the distance between the tips at the moment when it is increasing most rapidly?(Putnam Exam Question) Answer: Making assumption that both the hands moves at constant angular velocities, the answer is ## \sqrt{7} .## But don't you think this assumption is somewhat doubtful and wrong?

Similar threads

  • · Replies 2 ·
Replies
2
Views
2K
Replies
4
Views
2K
  • · Replies 9 ·
Replies
9
Views
2K
  • · Replies 8 ·
Replies
8
Views
2K
  • · Replies 5 ·
Replies
5
Views
989
  • · Replies 12 ·
Replies
12
Views
2K
  • · Replies 5 ·
Replies
5
Views
2K
  • · Replies 17 ·
Replies
17
Views
2K
  • · Replies 15 ·
Replies
15
Views
2K
  • · Replies 10 ·
Replies
10
Views
3K